LSAT and Law School Admissions Forum

Get expert LSAT preparation and law school admissions advice from PowerScore Test Preparation.

User avatar
 Dave Killoran
PowerScore Staff
  • PowerScore Staff
  • Posts: 5852
  • Joined: Mar 25, 2011
|
#41617
Complete Question Explanation
(The complete setup for this game can be found here: lsat/viewtopic.php?t=14535)

The correct answer is (D)

Answer choices (A), (B), and (C) can be eliminated by referring to the second and third rules. Answer choice (E) can be eliminated by the deduction made during the setup that placed P on floor 3. Answer choice (D) is thus correct. Answer choice (D) can also be confirmed very quickly by referring to the hypothetical created in question #14.

Get the most out of your LSAT Prep Plus subscription.

Analyze and track your performance with our Testing and Analytics Package.